Sei sulla pagina 1di 38

Geriatrics

Geriatrics
Jennifer P. Dugan, Pharm.D., BCPS
Kaiser Permanente Colorado Region
evergreen, Colorado

ACCP Updates in Therapeutics® 2012: The Pharmacotherapy Preparatory Review and Recertification Course

1-31
Geriatrics

Learning Objectives: C. Dementia is worsening.


1. Identify age-related pharmacokinetic and pharma- D. Urinalysis is falsely negative and should be
codynamic changes in older people. repeated.
2. Evaluate the pharmacotherapy regimens of older
people to support optimal physical and mental The following case pertains to questions 2 and 3.
function. J.T. is an 82-year-old community-dwelling woman with
3. Identify inappropriate medication prescribing in a history of stage III Parkinson disease, hypertension,
older people. and urinary incontinence (UI). She is receiving carbi-
4. Recommend appropriate pharmacotherapy for pa- dopa/levodopa, pramipexole, selegiline, tolterodine,
tients with dementia. diazepam, metoprolol, and hydrochlorothiazide. When
5. Evaluate the risks and benefits of the use of anti- she comes to your pharmacy to get her prescriptions,
psychotics (APs) (including atypical APs) in older she walks slowly with a cane, and she is stooped over.
patients with dementia.
6. Recommend appropriate interventions for patients 2. Which one of the following is the most important in-
suffering from behavioral symptoms related to tervention you can make to reduce her risk of falls?
dementia. A. Suggest to J.T. that her neurologist reassess
7. Identify the types of urinary incontinence and rec- her Parkinson disease medications.
ommend appropriate treatments. B. Advise J.T. that she should speak to her
8. Given a patient’s American Urology Association internist about discontinuing diazepam.
Symptom Index for benign prostatic hyperplasia, C. Suggest to J.T. that she see a physical therapist
recommend appropriate therapy. to institute a therapeutic exercise program.
9. Recommend appropriate analgesic therapy for old- D. Take J.T.’s BP to rule out postural effects
er patients with osteoarthritis. from treatment of her disease conditions.
10. Discuss risks and benefits of medication classes
used to treat rheumatoid arthritis. 3. While picking up her prescriptions at your phar-
macy, J.T. buys incontinence supplies. She confides
in you that her incontinence is more severe because
Self-Assessment Questions: she cannot get to the bathroom in time. On further
Answers and explanations to these questions questioning, you learn that J.T. believes her slowed
may be found at the end of this chapter. movement is a worsening problem. Which one of the
following is the best description of J.T.’s complaints?
1. R.J. is a 79-year-old woman you see in the clinic
A. Stress incontinence.
who has a history of mild dementia (Mini-Mental
B. Urge incontinence.
State Examination [MMSE] 23/30), hypertension,
C. Functional incontinence.
hypothyroidism, depression, and neuropathic pain.
D. Overactive bladder.
She has been taking atenolol 25 mg twice daily, le-
vothyroxine 125 mcg/day, amitriptyline 25 mg at
4. An 85-year-old woman is taking ferrous sulfate to
bedtime, and citalopram 20 mg/day. Her husband
treat an iron deficiency anemia. Changes in which
reports that she has been more confused in the past
one of the following pharmacokinetic properties
2 weeks. R.J.’s laboratory values are within normal
associated with aging can most affect this agent?
limits, including a normal thyroid-stimulating hor-
mone (TSH) and negative urinalysis. Her Geriatric A. Absorption.
Depression Scale (GDS) is 3/15. Her blood pres- B. Distribution.
sure (BP) is 145/85 mm Hg and heart rate (HR) is C. Metabolism.
65 beats/minute. Which one of the following is the D. Renal elimination.
most likely contributor to her confusion?
The following case pertains to questions 5–8.
A. Depression is not adequately treated.
W.G. is a 75-year-old African American man with coro-
B. Amitriptyline has anticholinergic effects.
nary artery disease, hypertension, type 2 diabetes melli-

ACCP Updates in Therapeutics® 2012: The Pharmacotherapy Preparatory Review and Recertification Course

1-32
Geriatrics

tus, chronic low back pain caused by osteoarthritis (OA) A. MMSE score of 20/30.
of the spine, peripheral neuropathy, and benign pros- B. Neuropsychiatric Inventory (NPI) score of
tatic hypertrophy (BPH). W.G. weighs 72 kg and is 71″ 75/120.
tall, and his serum creatinine (SCr) is 1.4 mg/dL. W.G.’s C. MMSE score of 10/30.
medications include metoprolol extended release 50 mg D. NPI score of 20/120.
once daily, lisinopril 20 mg/day, metformin 1000 mg 2
times/day, gabapentin 900 mg 3 times/day, glipizide ex- 10. After initiation of a cholinesterase inhibitor, which
tended release 10 mg/day, citalopram 20 mg/day, acet- of the following screenings is most indicated?
aminophen 1000 mg 3 times/day, Tylenol PM as needed A. Watch for new or worsening hypertension.
at night, and hydrocodone-acetaminophen 5 mg/500 mg B. Watch for new or worsening depression.
1 tablet as needed for pain every 4–6 hours. C. Watch for new or worsening hallucinations.
D. Watch for new or worsening urge
5. Given the pharmacokinetic changes in W.G., dose incontinence.
reduction is most indicated for which of the follow-
ing medications? The following case on L.M. pertains to questions 11–14.
A. Metoprolol. L.M. is a 92-year-old woman who is admitted to a nurs-
B. Metformin. ing facility after treatment of urosepsis at the local hos-
C. Glipizide. pital. Her diagnoses include hypertension, generalized
D. Gabapentin. anxiety disorder, dementia, transient ischemic attacks,
frequent urinary tract infections, OA, and inconti-
6. For W.G., which one of the following is most nence. L.M.’s medications include aspirin 81 mg/day,
strongly linked to his likelihood of experiencing a a multivitamin daily, amlodipine 5 mg/day, omeprazole
medication adverse event? 20 mg at night, sertraline 100 mg/day, cranberry cap-
A. Advanced age. sules 4 times/day, donepezil 10 mg at night, lorazepam
B. Use of many medications. 0.5 mg every 8 hours as needed for anxiety, quetiapine
C. Diagnosis of lower back pain. 25 mg 3 times/day, and the house bowel regimen. L.M.
D. Renal insufficiency. weighs 62 kg; her laboratory parameters are within nor-
mal limits except for an SCr of 1.5 mg/dL and blood
7. For W.G., which one of the following interventions urea nitrogen of 22 mg/dL. Her Tinetti fall risk assess-
is most important to avoid liver toxicity? ment is 9/28 (high risk), MMSE is 21/30, and GDS is
A. Decrease the dose of gabapentin. 1/15 (low risk).
B. Decrease the dose of glipizide.
C. Decrease the dose of acetaminophen from all 11. Given this patient’s medication regimen, which one
sources. of the following is L.M. at most increased risk of?
D. Discontinue metformin. A. Agitation.
B. Weight loss.
8. For W.G., which one of the following interventions C. Death.
is best to avoid urinary symptoms from BPH? D. Pain.
A. Discontinue Tylenol PM.
B. Discontinue glipizide. 12. L.M. is somnolent for most of the day and unable
C. Discontinue citalopram. to participate in activities on the unit. Which one of
D. Discontinue hydrocodone with the following interventions is the best approach?
acetaminophen. A. Reduce the dose of quetiapine over 1 month
and discontinue.
9. A patient with a recent diagnosis of Alzheimer dis- B. Discontinue donepezil.
ease is considering treatment with a cholinesterase C. Add a 7-day course of levofloxacin.
inhibitor. Which one of the following parameters D. Discontinue the cranberry supplement.
best justifies therapy initiation?

ACCP Updates in Therapeutics® 2012: The Pharmacotherapy Preparatory Review and Recertification Course

1-33
Geriatrics

13. L.M. is becoming combative during her personal


hygiene care in the morning, which is a change in
behavior for this patient. Which one of the follow-
ing is the best approach to this behavioral problem?
A. Treat L.M. with acetaminophen 500 mg 3
times/day for pain.
B. Avoid confrontation with L.M. and
discontinue her personal care.
C. Allow L.M. to sleep longer in the morning to
avoid fatigue.
D. Evaluate L.M. for reversible causes of
combative behavior.

14. The nursing staff wants to initiate analgesic treat-


ment for L.M.’s OA. Which one of the following is
the best therapy to recommend?
A. Ibuprofen 200 mg 4 times/day.
B. Acetaminophen 650 mg 3 times/day.
C. Tramadol 25 mg 3 times/day.
D. Oxycodone 5 mg 4 times/day.

15. S.C. is a 69-year-old woman with a history of rheu-


matoid arthritis admitted to a nursing facility af-
ter a 2-week hospitalization for sepsis. She reports
pain in her hands and wants treatment. Her labo-
ratory tests are within normal limits, except for
weight 47 kg, SCr 1.2, and glucose 160. Which one
of the following is the best treatment approach?
A. Start etanercept.
B. Start naproxen.
C. Start prednisone.
D. Start golimumab.

ACCP Updates in Therapeutics® 2012: The Pharmacotherapy Preparatory Review and Recertification Course

1-34
Geriatrics

I. Principles to Promote Optimal Medication Use in Older People

A. General Information About Aging in the United States


1. Aging is a process that converts healthy adults to frail adults with diminished reserves in physiologic
processes and increased vulnerability to many diseases and death.
a. The United States is projected to have 50 million people older than 65 years by 2020, 70 million
by 2030, and 80 million by 2050.
b. Even more dramatic increases are projected for people older than 85 years: 6 million by 2020, 8
million by 2030, and more than 18 million by 2050.
c. If a person survives to age 65, it is likely he or she will live an additional 13–20 years.
d. If a person survives to age 85, it is likely he or she will live an additional 6–7 years.
2. There is a large amount of heterogeneity in older people; some people live independently into their
90s and beyond, whereas others become frail and dependent at a younger age.
3. Measurement of aging with years of life is insensitive to the differences between older people.
4. Older adults are about 13% of the population, but they are also responsible for:
a. 34% of pharmaceutical spending
b. 36% of hospital stays
c. 40% of medication-related hospitalizations
d. 50% of medication-related deaths
5. At least $30 billion/year is spent on medication-related morbidity.

B. Optimal Pharmacotherapy in Older People


1. An optimal pharmacotherapy regimen is one in which patients receive effective medications to treat
conditions in the correct doses and dosage forms for the appropriate duration. Whenever possible, the
agent should be chosen to avoid drug-induced loss of patient function and drug-related problems.
2. The doses of many medications must be adjusted in older people because of age-associated changes in
drug pharmacokinetics and pharmacodynamics.
3. Drug pharmacokinetic changes

Table 1. Physiologic Changes with Age That May Change Drug Pharmacokinetics
Organ System Physiologic Change with Aging Resulting Effect on Pharmacokinetics
Gastrointestinal ↑ Stomach pH ↓ Absorption of some drugs and nutrients
↓ Gastrointestinal bloodflow ↓ In first-pass extraction/metabolism
Slowed gastric emptying Rate of absorption may be prolonged
Slowed gastrointestinal transit
Skin Thinning of dermis ↓ Or no change to drug reservoir formation with
Loss of subcutaneous fat transdermal formulation
Body composition ↓Total
 body water ↑ Volume of distribution and accumulation of lipid-
↓ Lean body mass soluble drugs
↑ Body fat ↓ Volume of distribution of water-soluble drugs
↓ Or unchanged serum albumin
↑ a1-Acid glycoprotein
Liver ↓ Liver mass No change in phase II drug metabolism
↓ Bloodflow to the liver ↓ Or no change in phase I metabolism
↓ Or no change in cytochrome P450 ↑ Half-life and ↓ clearance of drugs with a high first-pass
enzymes extraction/metabolism
Renal ↓ Glomerular filtration rate ↓ Renal elimination of many medications
↓Renal bloodflow ↑ Half-life of renally eliminated drugs and metabolites
↓ Tubular secretion
↓ Renal mass

ACCP Updates in Therapeutics® 2012: The Pharmacotherapy Preparatory Review and Recertification Course

1-35
Geriatrics

a. Absorption
i. Iron, vitamin B12, and calcium are decreased with hypo- or achlorhydria.
ii. Slower gastric emptying can increase risk of ulceration from aspirin or nonsteroidal anti-
inflammatory drugs (NSAIDs).
iii Most drugs are absorbed by passive diffusion without significant changes with aging.
iv. Transdermal formulations usually require a subcutaneous fat layer to form a drug reservoir
for absorption. Use with caution in individuals who are thin or cachetic.
b. Distribution
i. Lipid-soluble benzodiazepines such as diazepam have an increased half-life in older people.
ii. Highly albumin-bound drugs such as phenytoin may have a larger fraction of free (active)
drug.
iii. P-glycoprotein, an efflux transporter for several organs including the brain, is decreased with
aging, which may lead to higher brain concentrations of medications.
c. Metabolism
i. Morphine and propranolol clearance are substantially reduced because of a reduction in first-
pass metabolism.
ii. Changes in metabolism through phase I (oxidative) and cytochrome P450 (CYP) enzymes are
variable and confounded by age, sex, concomitant drugs, and genetics.
iii. Lorazepam, oxazepam, and temazepam are dependent on phase II metabolism and are less
affected by age-related changes in metabolism.
d. Elimination
i. Drugs eliminated through glomerular filtration must be dosed on the basis of the patient’s
estimated renal function.
ii. The Cockcroft-Gault equation is the most widely accepted method to estimate creatinine
clearance in the elderly.
iii. The four-variable Modification of Diet in Renal Disease equation is the most accepted
method to estimate glomerular filtration rate for diagnosing chronic kidney disease.

Table 2. Differences in CrCl Estimation with Accepted Formulas


BSA-Adjusted
Patient: 85-year-old person with Cockcroft-Gault Cockcroft-Gault Four-Variable MDRD
a serum creatinine of 1 mg/dL (mL/minute) (mL/minute/1.73m2) (chronic kidney disease stage)
5′4′′ white woman weighing 60 kg 39 41 56 mL/minute/1.73m 2 (stage 3)
5′4′′ black woman weighing 60 kg 39 41 68 mL/minute/1.73m 2 (stage 2)
5′10′′ white man weighing 75 kg 57 51 75 mL/minute/1.73m 2 (stage 2)
5′10′′ black man weighing 75 kg 57 51 91 mL/minute/1.73m 2
Cockcroft-Gault: CrCl = [(140 − age) × weight in kg]/[(72 × serum creatinine)] × 0.85 if female
[use actual weight if it is less than ideal body weight]
BSA-adjusted Cockcroft-Gault: (Cockcroft-Gault/patient’s BSA) × 1.73m 2/BSA
Four-variable MDRD: eGFR= 186 × [serum creatinine]-1.154 × [age]-0.203 × [0.742 if female] ×
[1.210 if African American]
BSA = body surface area; CrCl = creatinine clearance; eGFR = estimated glomerular filtration rate; MDRD = Modification of Diet in Renal
Disease (study).

4. Drug pharmacodynamic changes


a. Change in receptor number and/or affinity
i. More sensitive to benzodiazepines
ii. More sensitive to opioids

ACCP Updates in Therapeutics® 2012: The Pharmacotherapy Preparatory Review and Recertification Course

1-36
Geriatrics

iii. Increased response to warfarin


iv. Decreased response to β-blockers
v. More sensitive to extrapyramidal effects and tardive dyskinesia
b. Age-related changes in homeostatic response
i. Decreased baroreceptor response leading to orthostatic hypotension
ii. Sodium and water conservation
iii. Mobility and balance
5. Drug-related problems in the older person
a. Overuse of medications
i. May be the result of prescribing cascade
ii. Puts patient at higher risk of adverse drug effects and drug interactions
b. Underuse of medications
i. May include aspirin, angiotensin-converting enzyme (ACE) inhibitors, β-blockers, pain
medications, thrombolytics, antidepressants, and osteoporosis treatment
ii. Fear of falls may lead to underuse of warfarin, even in high-risk patients.
c. Nonadherence
i. Intentional nonadherence is related to a patient’s beliefs about treatment, including the need
for it and concerns about adverse effects.
ii. Unintentional nonadherence results from practical barriers such as misunderstanding
instructions, forgetfulness, or lack of resources to pay for treatment.
d. Use of inappropriate medications
e. Adverse drug events
f. Drug interactions
6. Function in the elderly
a. Quality of life and functional ability may be more important than duration of life.
b. Physical function is evaluated through one’s ability to complete basic and instrumental activities
of daily living (ADLs and IADLs).
c. Many elderly people are also screened for their risk of falls with instruments that evaluate gait and
balance (e.g., Tinetti Fall Risk, Timed Up and Go, Berg Balance).
d. Activities of daily living (ADLs)
i. Feeding
ii. Dressing
iii. Bathing
iv. Toileting
v. Ambulation
vi. Transferring
e. Instrumental activities of daily living (IADLs)
i. Cleaning
ii. Writing/reading
iii. Managing medications
iv. Shopping
v. Cooking
vi. Managing money
f. Mental function is evaluated with mental status examinations and additional screening for
depression.
g. It is important to consider the role of drugs on function in the elderly.
h. Examples of medications that can improve function in the elderly
i. Diuretics can improve dyspnea in heart failure.

ACCP Updates in Therapeutics® 2012: The Pharmacotherapy Preparatory Review and Recertification Course

1-37
Geriatrics

ii. Analgesics can decrease pain.


iii. Bronchodilators can improve dyspnea in chronic obstructive pulmonary disease.
iv. Antidepressants can decrease anhedonia.

Table 3. Function and Medications


Functional Domain Example Drugs That May Impair Function Interventions That May Promote Function
Mobility
Bone and muscle Glucocorticosteroids Vitamin D
integrity Long-term use of proton pump Calcium
inhibitors Bisphosphonates
Phenytoin Resistance exercises

Balance/dizziness/ Sedative-hypnotics Tai chi exercises


falls Antihypertensives Correction of vision problems
Long half-life benzodiazepines Environmental evaluation to remove
Large doses of benzodiazepines hazards
Tricyclic antidepressants (TCAs) Reduction in polypharmacy
Antipsychotics Addressing vitamin D or B12 deficiencies
Anticonvulsants

Extrapyramidal Typical antipsychotics


symptoms Metoclopramide
Mental state
Anticholinergic agents Reading, playing games, and working
Benzodiazepines puzzles
Pentazocine Exercise
Skeletal muscle relaxants Cholinesterase inhibitors
TCAs
Gastrointestinal antispasmodics
Diphenhydramine, antihistamines
Emotional state
Methyldopa Socialization
Reserpine Cognitive behavioral therapy
Anastrozole Health promotion interventions
Interferon a-2b Appropriate use of antidepressants
Tamoxifen
Continence
Anticholinergics Toileting schedules
α-Blockers Pelvic floor exercises
α-Agonists Proper diagnosis of incontinence with
Alcohol individualized treatment
Acetylcholinesterase inhibitors
Diuretics

ACCP Updates in Therapeutics® 2012: The Pharmacotherapy Preparatory Review and Recertification Course

1-38
Geriatrics

Patient Case
1. N.H. is an 85-year-old woman who resides in a nursing home. She weighs 65 kg. Her medical history is signifi-
cant for type 2 diabetes mellitus, hypertension, and moderate dementia, likely attributable to vascular changes.
Two years ago, she had a cerebrovascular accident, and 1 year ago, she sustained a right hip fracture after a
fall. Her regularly scheduled medications include glyburide 10 mg/day, lisinopril 10 mg/day, metformin 500
mg 2 times/day, donepezil 10 mg/day, aspirin 81 mg/day, and a multivitamin daily. Her as-needed medications
include zolpidem 5 mg/day as needed for sleep, meclizine 25 mg ½ tablet 3 times/day as needed for dizziness,
and the house bowel regimen. When recommending medication changes for this patient, which one of the fol-
lowing functional assessments is most important to evaluate?
A. IADLs.
B. Depression.
C. Gait and balance.
D. Pressure sores.

2. Your further evaluation of N.H. reveals that she has not used any of her as-needed medications in 2 months.
In addition, her laboratory results reveal the following: fasting plasma glucose 90 mg/dL, sodium 138
mEq/L, potassium 4.5 mEq/L, chloride 102 mEq/L, CO2 25 mEq/L, blood urea nitrogen 30 mg/dL, SCr 1.8
mg/dL, and TSH 4.0 mU/L. Which one of the following pharmacokinetic parameters is most likely to be
changed in N.H.?
A. Oral absorption.
B. Distribution.
C. Metabolism.
D. Renal excretion.

3. Based on your assessment of age- and disease-related changes in N.H., which one of the following areas of
pharmacotherapy is best to address first?
A. Diabetes treatment.
B. Alzheimer disease treatment.
C. Hypertension treatment.
D. Stroke prevention.

7. Potentially inappropriate medications


a. Presence of inappropriate medication use in the elderly is often used as a quality assurance
measure.
b. Measures often include both medications/classes to avoid in general and medications to be
avoided in certain diseases or conditions.
c. Beers List
i. Initially identified drugs with a high likelihood of causing problems in long-term care
residents
ii. Developed through a consensus criteria process
iii. Refined during the past 10 years to improve applicability, including community-dwelling
elderly
iv. Examples include long half-life benzodiazepines, certain antihistamines, skeletal muscle
relaxants, drugs with anticholinergic effects, some narcotics, and some NSAIDs.
v. Limitations of using Beers List

ACCP Updates in Therapeutics® 2012: The Pharmacotherapy Preparatory Review and Recertification Course

1-39
Geriatrics

(a) Does not consider appropriate medications used inappropriately


(b) Does not consider other drug-related problems
(c) Addresses only medications about which a consensus could be reached
(d) Conflicting evidence exists on how the use of medications on Beers List affects function
and mortality.

Patient Case (N.H. from above)


4. To maintain and improve function in N.H., which one of the following interventions is best to implement?
A. Add a calcium and vitamin D supplement.
B. Add simvastatin 10 mg/day.
C. Add warfarin.
D. Assess for incontinence and treat with anticholinergic agents.

8. Falls
a. Pharmacists are frequently consulted to review medication regimens for high-risk patients or
patients who have experienced a fall.
b. Psychoactive medications (sedative-hypnotics, anxiolytics, antipsychotics, antidepressants)
should be minimized or withdrawn if possible.
c. Polypharmacy by itself is a risk factor for falls; evaluate medications for indication and risks/
benefits.
d. Vitamin D deficiency should be assessed for and corrected, and vitamin D supplementation of
800 IU/day can be considered for all older adults at high risk of falls.
e. Orthostatic hypotension is associated with a greater risk of falls.
f. Hyponatremia is a risk factor for falls, and it may be worsened by diuretics, selective serotonin
reuptake inhibitors (SSRIs), and ACE inhibitors.
g. Vitamin B12 deficiency is also a risk factor falls. B12 absorption may be impaired by proton pump
inhibitors, histamine-2 blockers, and potassium supplements.
9. Creation of an optimal pharmacotherapy regimen
a. Use as few medications as possible.
b. Every drug used should have evidence-based literature to support treatment of the particular
indication.
c. All drugs should be screened to avoid drug interactions (e.g., drug, disease, food, and
supplements).
d. Evaluate the drug dose for possible adjustments caused by pharmacokinetic changes.
e. Evaluate the drug for proper therapy duration.
f. Avoid duplicate therapy.
g. Ensure it is affordable for patient.
h. Evaluate unexplained patient symptoms to rule out drug adverse effects.
i. Ensure that treatment goals are appropriate given patient age and frailty.
i. ACCF/AHA 2011 expert consensus document on hypertension in the elderly indicates a goal
systolic BP less than 140 mm Hg for those 79 years or younger is appropriate; but for those
80 years and older, 140–145 mm Hg, if tolerated, can be acceptable.
ii. The American Geriatrics Society and American Diabetes Association recommend that target
A1c levels be individualized for patients. For frail, older adults; people with a life expectancy
of less than 5 years; and others in whom the risks of intensive glycemic control appear to
outweigh the benefits, a less-stringent target such as less than 8% is appropriate.

ACCP Updates in Therapeutics® 2012: The Pharmacotherapy Preparatory Review and Recertification Course

1-40
Geriatrics

II. DEMENTIA

A. Epidemiology
1. Dementia affects 4–5 million people in the United States.
2. Disease is strongly associated with advanced age.
a. 2% prevalence in 65 year olds
b. Up to 30% prevalence in 85 year olds

B. Definitions
1. Mild cognitive impairment
a. Transitional stage between normal aging and early dementia
b. 5%–25% will progress to develop dementia.
2. Delirium
a. Mental status change with recent onset and fluctuation
b. Includes two of the following:
i. Misinterpretation, illusions, hallucinations
ii. Incoherent speech at times
iii. Disturbance in sleep-wake cycle
iv. Change in psychomotor activity
c. Usually related to medical illness or medications and is reversible
3. Dementia
a. Progressive, irreversible decline in mental function, marked by memory impairment
b. Not a normal part of aging
c. Symptoms include functional disability, cognitive impairments, and behavioral and psychological
symptoms.

C. Reversible Causes of Mental Status Change


1. Alcohol abuse
2. Normal pressure hydrocephalus
3. Thyroid dysfunction
4. Dehydration and electrolyte disturbance(s)
5. Vitamin B12 deficiency
6. Depression
7. Medications
a. Medications with anticholinergic properties
b. Opioids
c. Corticosteroids
d. Benzodiazepines and other sedative-hypnotics
e. Antiparkinsonian medications

D. Most Common Types of Dementia


1. Alzheimer disease
2. Lewy body dementia
3. Vascular dementia
4. Frontotemporal lobe dementia (Pick disease)
5. Mixed dementia

ACCP Updates in Therapeutics® 2012: The Pharmacotherapy Preparatory Review and Recertification Course

1-41
Geriatrics

Patient Case
5. An 85-year-old woman is assessed at a memory loss clinic to determine the cause of her dementia. Her most
recent score on the MMSE is 24/30. Present diagnoses include Parkinson disease, hypothyroidism, and OA
of both knees. She has had these conditions for more than 10 years, is stable, and is independent in her ADLs.
Her present medications include carbidopa/levodopa continuous release, trihexyphenidyl, celecoxib, levothy-
roxine, docusate, and bisacodyl. Which one of the following medication changes is best to consider first?
A. Add donepezil 5 mg/day.
B. Slow dosage reduction of carbidopa/levodopa.
C. Slow dosage reduction and discontinue trihexyphenidyl.
D. Replace celecoxib with acetaminophen.

E. Pathophysiology of Alzheimer Disease


1. Precise cause is not known.
2. Several interrelated causative factors
a. Genetics and the apolipoprotein E4 allele
b. Environment
c. Accumulation of β-amyloid
d. Tau
e. Inflammation
f. Neurotransmitter deficiency (acetylcholine, norepinephrine, serotonin)
3. Common assessment instruments used for Alzheimer disease
a. Mental state
i. Folstein MMSE
(a) 30-point scale, with higher scores indicating better mental functioning
(b) If patient is untreated, score usually decreases 3 or 4 points a year.
ii. Alzheimer Disease Assessment Scale – Cognitive (ADAS-cog)
(a) 70-point scale, with lower scores indicating better mental functioning
(b) Used in most research
iii. St. Louis University Mental Status Examination (SLUMS)
(a) 30-point scale, with higher scores indicating better mental functioning
(b) Similar to MMSE; adjust for education status
iv. Montreal Cognitive Assessment ( MOCA): 30-point scale, with higher scores indicating
better mental functioning
v. Mini-Cog Assessment
(a) Combines a clock-drawing test with a three-item recall test
(b) Used in clinical practice because of ease of administration (less than 5 minutes)
b. Function
i. Alzheimer Disease Cooperative Study Activities of Daily Living Inventory (ADCS-ADL):
54-point scale, with higher scores indicating better function
ii. Functional Assessment Staging (FAST)
(a) 7-point scale
(b) 1 indicates normal adult, 7 indicates patient requires total care
iii. Various scales to measure IADLs
c. Global assessment: Clinician Interview–Based Impression of Change (CIBIC-Plus) Plus
Caregiver Input
i. 7-point scale
ii. 1 shows marked improvement, 7 shows marked deterioration, 4 is no change

ACCP Updates in Therapeutics® 2012: The Pharmacotherapy Preparatory Review and Recertification Course

1-42
Geriatrics

F. Clinical Presentation and Classification

Table 4. Stages of Alzheimer Disease


Folstein Mini-
Mental State
Examination (of 30) Examples of Cognitive Loss Examples of Functional Loss
Mild 20–24 Some short-term memory loss; Loss of IADLs such as laundry,
word-finding problems housekeeping, and managing
medications; may get lost in familiar
places
Moderate 10–19 Disorientation to time and place, Needs assistance with ADLs such as
inability to engage in activities bathing, dressing, and toileting
and conversation
Severe < 10 Loss of speech and ambulation, Dependency in basic ADLs such as
incontinence of bowel and bladder feeding oneself; often requires around-
the-clock care
ADLs = activities of daily living; IADLs = instrumental activities of daily living.

G. Treatment
1. Goals
a. Maintain cognition or slow progression of decline.
b. Maintain function.
2. Nonpharmacologic therapy
a. Education, particularly directed toward the caregiver
b. Reduction in environmental triggers
c. Optimal management of comorbid diseases
3. Pharmacologic therapy

ACCP Updates in Therapeutics® 2012: The Pharmacotherapy Preparatory Review and Recertification Course

1-43
Geriatrics

Table 5. Comparison of Drugs for the Treatment of Alzheimer Disease


Maintenance Dosage Pharmacologic
Starting Dose Dose Forms Properties Comments
Cholinesterase inhibitors
Donepezil 5 mg/day 10 mg/day 5-, 10-, and Acetylcholinesterase Labeled for mild-moderate
(Aricept); 23-mg tablets, inhibitor; metabolized and severe Alzheimer
donepezil orally May also 5- and 10- in part by CYP2D6 and disease. Risk of
disintegrating increase to 23 mg orally CYP3A4 bradycardia and syncope
tablets (Aricept mg/day disintegrating is increased with all CIs
ODT) tablets
Rivastigmine 1.5 mg 2 3–6 mg 2 times/ 1.5-, 3-, 4.5-, Acetyl and butyl- Labeled for mild-moderate
(Exelon); times/day day and 6-mg cholinesterase inhibitor. Alzheimer disease as
rivastigmine capsules; 2 Cholinergic adverse well as mild-moderate
patch (Exelon mg/mL of oral effects of nausea, dementia with Parkinson
Patch) solution vomiting, and diarrhea disease
seem more intense than
9-mg patch 18-mg patch Transdermal with other CIs
(delivers 4.6 (delivers 9.5 patch delivers
mg/day) mg/day) drug over 24
hours
Galantamine 4 mg 2 times/ 8–12 mg 2 4-, 8-, 12-mg A selective competitive, Preferable to administer
(Razadyne); day times/day or tablets; 8-, reversible inhibitor of with food. Syncope is
galantamine 8–24 mg ER 16-, 24-mg acetylcholinesterase seen more often at higher
extended release once daily extended- and a nicotine receptor dosages
(Razadyne ER) release modulator. Metabolized
capsules; 4 in part by CYP2D6 and
mg/mL of oral CYP3A4
solution
Glutamatergic therapy
Memantine 5 mg once 10 mg 2 5- and 10-mg N-methyl-d-aspartate Labeled for use in patients
(Namenda) daily times/day tablets; 2 mg/ receptor antagonist with moderate to severe
mL of oral that blocks glutamate Alzheimer disease; may
solution transmission be used in combination
with acetylcholinesterase
inhibitors. In general, well
tolerated but confusion
sometimes seen
CI = cholinesterase inhibitor; CYP = cytochrome P450; ER = extended release.

Patient Case
6. An 87-year-old man with Alzheimer disease began therapy with rivastigmine. He has been titrated up to
rivastigmine 6 mg 2 times/day. His family notes improvement in his functional ability but reports that he is
experiencing nausea and vomiting that seem related to rivastigmine. Which one of the following is the best
course to take?
A. Advise the patient to take his drug with an antacid.
B. Add prochlorperazine 25 mg by rectal suppository with each rivastigmine dose.
C. Discontinue rivastigmine and initiate memantine 5 mg twice daily.
D. Change rivastigmine to the daily patch that delivers 9.5 mg/day.

ACCP Updates in Therapeutics® 2012: The Pharmacotherapy Preparatory Review and Recertification Course

1-44
Geriatrics

4. Review of consensus treatment guidelines from the American College of Physicians and American
Academy of Family Physicians, and a Guide to Dementia Diagnosis and Treatment the American
Geriatrics Society
a. Therapy initiation
i. For patients with mild-moderate Alzheimer disease, begin therapy with a cholinesterase
inhibitor when the benefits of treatment outweigh the risks and are consistent with individual
patient goals.
ii. Evidence from high-quality studies often shows statistically significant differences in
measures of cognition, global assessment, and, occasionally, ADL function.
iii. Often, the statistically significant differences found in the high-quality research are not
deemed clinically meaningful.
iv. Only 10%–25% of patients taking cholinesterase inhibitors may show modest global
improvement, but more patients may have less rapid cognitive decline. There is no way to
determine which individuals may show clinically meaningful improvement before initiating
therapy.
v. There is no evidence that any one cholinesterase inhibitor is better than another.
vi. Results for memantine are similar to those for cholinesterase inhibitors: Statistically
significant but questionable clinically meaningful changes. Memantine is labeled for use
in patients with moderate-severe Alzheimer disease. It has been studied in randomized
controlled trials as monotherapy and in combination with donepezil.
b. Therapy duration
i. Expected decline without treatment is about 4 points/year on MMSE/SLUMS/MOCA.
Evaluate 3–6 months after treatment initiation for stabilization (prevention of decline) or
improvement using repeat testing and caregiver input.
ii. Conflicting evidence exists regarding therapy discontinuation, should a patient decline or
become unresponsive to treatment.
iii. When prevention of decline is no longer a therapeutic goal for a patient, consider
discontinuing drug treatment.
iv. A taper is recommended if a patient is at a higher dose.
v. Rebound agitation may occur for the first 1–2 weeks.

Patient Case
7. R.A. is a 75-year-old woman with Alzheimer disease who has been treated with donepezil 10 mg/day for
about 3 years. When she initiated therapy, her MMSE was 21/30, and her present MMSE is 17/30. R.A. is
living at home with her husband. She cannot perform most IADLs but can perform most ADLs with cueing.
R.A.’s husband asks about changing her drug treatment to help maintain her function. Which one of the fol-
lowing is the best course of action?
A. Change her treatment from donepezil to rivastigmine.
B. Stop donepezil.
C. Add memantine 5 mg/day.
D. Add vitamin E 400 units 2 times/day.

ACCP Updates in Therapeutics® 2012: The Pharmacotherapy Preparatory Review and Recertification Course

1-45
Geriatrics

III. Behavioral symptoms of dementia

A. Epidemiology
1. More than 50% of people with dementia have significant behavioral and psychological symptoms.
2. These symptoms are associated with high rates of disability and functional decline, poor health
outcomes, physical injury, increased rates of nursing home placement, greater use of emergency
services, and difficulty with nurse retention.

B. Types of Behavioral Symptoms


1. Psychosis
a. Delusions
b. Paranoia
c. Hallucinations
2. Agitation
a. Aggression and combativeness
b. Hyperactivity (wandering and pacing)
c. Hyper-vocalization
d. Disinhibition

C. Measurement of Behavioral Symptoms


1. Neuropsychiatric Inventory
a. 144-point scale
b. Higher scores indicate severer behavioral problems.
2. Behavioral Pathology in Alzheimer’s Disease (BEHAVE-AD)
a. 75-point scale
b. Higher scores indicate more behavioral problems.
3. Cohen-Mansfield Agitation Inventory (CMAI)
a. 203-point scale
b. Higher scores indicate more behavioral problems.
4. Scales are not always used; generally, clinicians identify “target behavior.”

D. Management of Behavioral Symptoms


1. Determine the cause.
a. Pain
b. Constipation
c. Hunger and dehydration
d. Depression
e. Fear
f. Anxiety
g. Loss of sleep
h. Infection
i. Drug adverse effects
2. The 5 Rs
a. Reassess
b. Reconsider
c. Rechannel
d. Redirect or remove
e. Reassure

ACCP Updates in Therapeutics® 2012: The Pharmacotherapy Preparatory Review and Recertification Course

1-46
Geriatrics

3. Nonpharmacologic interventions
a. Educate caregivers.
b. Patient interventions
i. Correct underlying factors that may contribute to agitation.
ii. Establish a routine.
iii. Orientation aids
iv. Segment task.
v. Music, pet, and other forms of behavioral therapy
c. Environmental interventions
i. Improve safety.
ii. Appropriate lighting
iii. Appropriate amount of sensory stimulation
iv. Increase in personal space

Table 6. Examples of Nonpharmacologic Interventions


Behavior Causes Management
Agitation Pain, discomfort Assess and manage pain, constipation, possible infection
Physical illness Evaluate medically and treat
Overstimulation Reduce noise and stress, limit TV, avoid crowding
Paranoia Forgot where object placed Offer to help find; have more than one of same object
Misinterpreting actions or words Do not argue or try to reason, do not take personally; distract
Change in environment Familiarize, reassure, set routine
Insomnia Depression Treat with antidepressant
Less need for sleep Later bedtime, more exercise, monitor daytime napping

4. Pharmacologic interventions
a. Cholinesterase inhibitors
i. Cochrane comprehensive review showed a small improvement in the NPI with donepezil and
galantamine after several months.
ii. A more recent randomized controlled trial did not show improvement in the NPI with
donepezil treatment.
iii. An increase in agitation may occur with initiation of cholinesterase inhibitor treatment.
iv. Cholinesterase inhibitors are first line for psychosis in Lewy body dementia.
b. Atypical antipsychotics (APs)
i. No AP approved by the U.S. Food and Drug Administration (FDA) for the treatment of
dementia-related psychosis or agitation.
ii. No clear standard on when to use, but generally should be reserved for patients who are
aggressive, pose a danger to self or others, have symptoms that are interfering with function,
or experience delusions or hallucinations that are distressing to the patient. This should be
done only if the patient has not responded to a comprehensive nonpharmacologic plan or if
the patient requires emergency intervention for further workup.
iii. Cochrane comprehensive review and recent meta-analysis suggests olanzapine and
risperidone have the most evidence for use in psychosis and aggression.
iv. APs have a high rate of adverse effects such as sedation, orthostasis, dose-dependent
movement disorders, and increased risk of stroke and mortality.
v. Conventional APs do not appear to be any safer.
vi. Increased risk of death validated in several reviews (number need to harm =87)

ACCP Updates in Therapeutics® 2012: The Pharmacotherapy Preparatory Review and Recertification Course

1-47
Geriatrics

vii. Discuss risks and benefits with the patient’s family.


viii. Use quetiapine if the patient has comorbid Parkinson disease or Lewy body dementia.
ix. Use for the shortest time possible and taper to discontinue.
c. Other medications
i. Antidepressants
(a) Especially useful for patients with underlying anxiety or depression
(b) SSRIs usually first line (except paroxetine because of anticholinergic effects)
(c) Start at low dose to avoid increasing agitation.
(d) Watch for GI symptoms and hyponatremia.
ii. Valproic acid/carbamazepine
(a) May be useful for target symptoms of labile or irritable mood, impulsivity, or
combativeness
(b) Conflicting evidence to support
(c) Use lower doses; drug levels do not correlate with efficacy in dementia.
iii. Memantine
(a) Minimal evidence that patients taking memantine experience less agitation
(b) Awaiting the results of clinical trials for the treatment of agitation
iv. Benzodiazepines are not indicated for the treatment of agitation and frequently cause
disinhibition.

Patient Cases
8. You are evaluating the medication profile of an 87-year-old female nursing home resident. She resides in
a secure advanced dementia unit. Her medical history includes dementia probably caused by Alzheimer
disease, Parkinson disease, and OA. She requires assistance with all ADLs including total assistance with
bathing and dressing and help with feeding. She ambulates with the assistance of a four-wheeled walker.
Her medication regimen includes donepezil 10 mg/day, memantine 10 mg 2 times/day, carbidopa/levodopa
25/100 mg 4 times/day, oxybutynin extended release 5 mg/day, and a multivitamin supplement daily. The
patient’s MMSE score is 5/30, and her GDS is 4/15. When reviewing the nursing notes, there are several ref-
erences to the patient’s continuously crying out, “Help me, help me.” Which one of the following additional
assessment tools is most necessary in assessing this patient?
A. The Brief Psychiatric Rating Scale.
B. Functional Assessment Staging.
C. An evaluation of incontinence.
D. Framingham Risk Assessment.

9. Which one of the following changes would be best to reduce inappropriate medications?
A. Change carbidopa/levodopa to a continuous-release formulation.
B. Discontinue oxybutynin.
C. Discontinue memantine.
D. Reduce dose of donepezil.

10. This patient is medically assessed, and reversible causes of her hyper-vocalization are ruled out. Which one
of the following represents the best approach to treating her behavioral symptoms?
A. Implement a behavioral approach.
B. Add valproic acid.
C. Add quetiapine.
D. Add citalopram.

ACCP Updates in Therapeutics® 2012: The Pharmacotherapy Preparatory Review and Recertification Course

1-48
Geriatrics

Table 7. Analysis of Significant Research on AP Use in the Elderly


Study Study Parameters Outcome Comments
Research evaluating drug efficacy in patients with behavioral disorders related to dementia
Ballard, Cochrane Meta-analysis to assess Evidence suggests that Authors’ conclusion: “Despite
Database 2008 whether evidence supports risperidone and olanzapine are the modest efficacy, the
the use of atypical APs for useful in reducing aggression significant increase in adverse
the treatment of aggression, and that risperidone reduces events confirms that neither
agitation, and psychosis psychosis, but both are risperidone nor olanzapine
in people with Alzheimer associated with serious adverse should be used routinely to
disease cerebrovascular events and treat dementia patients with
extrapyramidal symptoms aggression or psychosis unless
there is severe distress or risk of
physical harm to those living and
working with the patient.”
Schneider, N Engl Double-blind placebo- No difference in the time to The dose of quetiapine was
J Med 2006 controlled trial of 421 discontinuation for any of about one-half to one-fourth of
patients with Alzheimer the three APs. A high rate what is generally used in this
(CATIE-AD) disease and behavioral of discontinuation of APs patient population. Overall, the
symptoms randomized to occurred because of patient median time to discontinuation
olanzapine, quetiapine, or intolerance (parkinsonism, was 5–8 weeks, and about half
risperidone. Main outcome sedation, confusion, and of the patients had discontinued
was time to discontinuation weight gain) by 8 weeks
of AP for any reason
Sink, JAMA 2005 Double-blind, randomized Evidence to support the In general, minimal evidence
trials (or meta-analyses of efficacy of typical and atypical to support drug treatment of
the above) were evaluated APs is weak with significant neuropsychiatric symptoms
for APs, antidepressants, adverse events; treatment caused by dementia. The studies
CIs, and mood stabilizers with antidepressants improves evaluated often had inconsistent
used among patients with depression but not other results among differing measures
behavioral symptoms neuropsychiatric symptoms; of behavior
caused by dementia no clear evidence to support
valproate or carbamazepine;
small but consistent results
with CIs
Research involving risk of mortality in patients with dementia who received AP treatment
Schneider, JAMA Meta-analysis of 15 Overall odds ratio of death of Increase in mortality was
2005 randomized trials 1.54 observed for aripiprazole,
quetiapine, olanzapine, and
3353 patients with (confidence interval 1.06–2.23) risperidone. Trial duration was
Alzheimer disease with from 8 to 22 weeks
agitation were randomly p=0.02
assigned to placebo or AP
Wang, N Engl J Retrospective cohort The relative risk of death after Both types of APs are associated
Med 2006 evaluation of more than AP drug administration was with an increased risk of death in
22,000 older people who higher with conventional APs older people
received either conventional vs. atypical APs
or atypical APs

ACCP Updates in Therapeutics® 2012: The Pharmacotherapy Preparatory Review and Recertification Course

1-49
Geriatrics

Table 7. Analysis of Significant Research on AP Use in the Elderly (continued)


Study Study Parameters Outcome Comments
Ballard, Lancet A double-blind trial of At 12 months, the cumulative Most patients were receiving
2009 165 patients in long-term survival was 70% in those who either risperidone or haloperidol
care who were receiving remained on APs vs. 77% in treatment
(DART-AD) an AP were randomized those who were randomized
to discontinue the AP or to placebo. The survival
to continue with their AP difference in the placebo group
therapy was more pronounced at 24 and
36 months
Maher, JAMA A systematic review on Among 14 placebo-controlled Benefit was considered of
2011 the efficacy and safety trials of elderly patients with marginal clinical significance
of atypical antipsychotic dementia reporting a total
medications for use in off- global outcome score that
label conditions includes symptoms such as
psychosis, mood alterations,
and aggression, small but
statistically significant effects
ranging from 0.12 and 0.20
were observed for aripiprazole,
olanzapine, and risperidone.
NNH for death was 87
AP = antipsychotic; CIs = cholinesterase inhibitors; NNH = number needed to harm.

IV. Urinary Incontinence

A. Epidemiology
1. In community-dwelling elderly individuals, UIs affect about 38% of women and 17% of men.
2. More than half of nursing home residents suffer from UIs.
3. Although the prevalence of UIs is correlated with age, UIs are not part of normal aging.
4. UI is a costly condition, with estimates of more than $5 billion annually spent by long-term care
institutions.

B. Physiology of the Genitourinary System and Urination


1. Successful urination is accomplished through a complex system involving the central nervous system,
spinal cord, peripheral nerves, and lower urinary tract.
a. In the lower urinary tract, the bladder is regulated by the sympathetic nervous system through
acetylcholine interactions with muscarinic receptors.
i. Stimulation of the sympathetic nervous system allows detrusor muscle relaxation and bladder
filling.
ii. The M3 receptor is the predominant muscarinic receptor in the bladder, and its stimulation
results in bladder contraction to enable emptying.
b. The internal urethral sphincter is regulated by the parasympathetic nervous system.
i. During bladder filling, the urethral sphincter is contracted through inhibition of the
parasympathetic nervous system.
ii. When the bladder is full, signals from the brain result in parasympathetic stimulation, and
urethral sphincter relaxation occurs.

ACCP Updates in Therapeutics® 2012: The Pharmacotherapy Preparatory Review and Recertification Course

1-50
Geriatrics

c. Urination results when the bladder intravesical pressure exceeds resistance from the urethra and
urethral sphincter.
2. Aging effects on the genitourinary system
a. Decrease in bladder elasticity
b. Reduction in bladder capacity
c. More frequent voiding
d. Genitourinary changes related to sex
i. Women have a decline in bladder outlet and urethral resistance because of a loss of estrogen,
which weakens the pelvic musculature.
ii. Men may experience a decrease in urine flow rate and instability of the detrusor muscle from
prostatic enlargement.
3. Causes/risk factors for UI: An evaluation of UIs is important to correct any reversible underlying
conditions. The following mnemonic is sometimes used, spelling the word DIAPERS: Delirium,
Infection, Atrophic vaginitis and urethritis, Psychiatric disorders, Excessive urine output, Restricted
mobility, and Stool impaction.
4. Types of UIs

Table 8. Common Types of UIs, Drug-Induced Causes, and Treatment


Type of Drug-Induced
Incontinence Description Causes Drug Treatment Comments
Urge or Loss of moderate Cholinergic agents Antimuscarinic/ Anticholinergic agents are
overactive amounts of urine that stimulate the anticholinergic agents first-line therapy. Differences
bladder with an increase bladder such as Oxybutynin, tolterodine, in muscarinic receptor–
in need to void. bethanechol and fesoterodine, trospium, blocking exist, but clinical
Detrusor instability cholinesterase solifenacin, darifenacin differences in efficacy
can be caused by inhibitors between these agents has not
CNS damage from a been shown.
stroke
Tricyclic TCAs are not preferred
antidepressants for use in the elderly
Imipramine, doxepin, because of their nonspecific
nortriptyline, desipramine anticholinergic profile
Stress Loss of urine α-Blockers such as α-Adrenergic agonists Efficacy evidence is limited
incontinence with increased prazosin decrease Pseudoephedrine, with the α-adrenergic agonists
abdominal pressure urethral sphincter phenylephrine and estrogen for stress
(sneezing, coughing, tone UI. Surgery to support the
etc.). Stress UI is Topical estrogens bladder and bladder neck is
more common in Conjugated estrogen highly effective
postmenopausal vaginal cream or
women and in men estradiol vaginal Not FDA label approved
s/p prostatectomy insert/ring for stress UI but has best
evidence of the medication
Serotonin/ options; adverse effects
norepinephrine (including nausea, dry mouth,
reuptake inhibitor and fatigue) may limit its
Duloxetine usefulness

ACCP Updates in Therapeutics® 2012: The Pharmacotherapy Preparatory Review and Recertification Course

1-51
Geriatrics

Table 8. Common Types of UIs, Drug-Induced Causes, and Treatment (continued)


Type of Drug-Induced
Incontinence Description Causes Drug Treatment Comments
Overflow Loss of urine Anticholinergic α-Adrenergic Adverse effects vary
incontinence because of excessive agents, calcium antagonists: depending on selectivity
bladder volume channel blockers, (outlet obstruction) to receptors in the bladder
caused by outlet and opioids Alfuzosin, tamsulosin, and/or prostate (alfuzosin,
obstruction or decrease detrusor silodosin, doxazosin, silodosin, and tamsulosin are
an acontractile muscle contractions terazosin, prazosin more specific)
detrusor. A postvoid
residual (PVR) is Add-on therapy For advanced BPH or
often high (> 300 with 5-α-reductase refractory symptoms
mL), indicating inhibitors:
incomplete Finasteride, dutasteride
emptying
Cholinomimetics: Stimulates the detrusor
Bethanechol muscle

Phosphodiesterase-5 5 mg once daily approved for


Inhibitors: BPH
Tadalafil
Functional Inability to reach Sedative-hypnotics Remove barriers and Consider interventions to
incontinence the toilet because of and other sedating obstacles, provide remove any potential cause
mobility constraints drugs that may schedules or prompted
cause confusion; toileting; assistance may
diuretics may be required to transfer
increase voiding on/off commode
Mixed UI that has more Focus on symptoms that Consider treatments for
incontinence than one cause; dominate individual components (i.e.,
usually stress and stress and urge)
overactive bladder
BPH = benign prostatic hypertrophy; CNS = central nervous system; FDA = U.S. Food and Drug Administration; s/p = status post; TCA =
tricyclic antidepressant; UI = urinary incontinence.

5. Nonpharmacologic interventions to consider:


a. Pelvic floor exercises (Kegel exercises)
b. Bladder training – Used to increase time between voiding, especially in urge incontinence
c. Biofeedback – For both stress and urge incontinence, to teach pelvic floor exercises and urge
inhibition
d. Scheduled/timed voiding
e. Avoid aspartame, spicy and citrus foods, caffeine, carbonated beverages
f. Pessaries/bladder neck support prostheses – primarily for stress incontinence

ACCP Updates in Therapeutics® 2012: The Pharmacotherapy Preparatory Review and Recertification Course

1-52
Geriatrics

Table 9. Comparison of Adverse Effects from Urinary Antimuscarinic Agentsa


Drug Dry Mouth (%) Constipation (%) Dizziness (%)
Oxybutynin 88 32 38
Oxy ER/XL 68 9 11
Oxy TDS 10 5 4
Oxy gel 8 1 3
Tolterodine 50, 39 10, 10 4, 3
Fesoterodine 99 14 2
Trospium 33 11 ?
Solifenacin 34 19 1
Darifenacin 59 28 0
a
Treatment of overactive bladder in women. AHRQ Publication No. 09-E017. August 2009.
ER/XL = extended release.

Patient Case
11. A 75-year-old woman reports urinary urgency, frequency, and loss of urine when she cannot make it to the
bathroom in time. She also wears a pad at night that she changes 2 or 3 times because of incontinence. Her
medical history is significant for Alzheimer disease (MMSE 23), osteoarthritis, and hypothyroidism. A
urinalysis is negative, her examination is normal, and postvoid residual (PVR) is normal (less than 100 mL).
Which one of the following interventions would be best at this time?
A. Bethanechol.
B. Pelvic floor exercises plus estrogen vaginal cream.
C. Darifenacin.
D. Oxybutynin.

V. Benign prostatic hypertrophy

A. Epidemiology
1. Prostatic hypertrophy usually develops after age 40.
2. By age 60, half of all men have BPH; by age 85, 90% have BPH.
3. Growth of the prostate gland leads to narrowing of the urethra and obstruction of urinary flow.

B. Pathophysiology and Clinical Presentation


1. Pathophysiology is not entirely understood; however, type II 5-α-reductase facilitates the conversion
of testosterone to dihydrotestosterone (DHT), resulting in prostate growth.
2. Lower urinary tract symptoms (LUTS)
a. Voiding (obstructive) symptoms: Decreased force of stream, hesitancy, straining, dribbling,
prolonged voiding, incomplete voiding, retention
b. Storage (irritative) symptoms: Urinary urgency, frequency, nocturia, small voided volume, urge
incontinence, dysuria
3. The American Urological Association Symptom Index (AUASI) can help determine severity and
appropriate treatment. The index consists of seven questions evaluating the severity of LUTS on a 0–5
scale. Higher numbers indicate severer symptoms.

ACCP Updates in Therapeutics® 2012: The Pharmacotherapy Preparatory Review and Recertification Course

1-53
Geriatrics

C. Evaluation of BPH
1. All patients should undergo a medical history, digital rectal examination, and urinalysis.
2. Selected individuals should be further assessed with the prostate-specific antigen (PSA) test.
3. Patients with mild symptoms on the AUASI (total score of 0–7) should undergo watchful waiting.
4. Patients with moderate LUTS on the AUASI (8–19) can consider medical therapy.
5. Patients with severe LUTS (20 or higher on the AUASI) and those with the following problems should
be assessed for surgery.
a. Persistent gross hematuria
b. Bladder stones
c. Recurrent urinary tract infections
d. Renal insufficiency
6. Assess for medications that may exacerbate BPH symptoms.
a. Adrenergic agonists (decongestants) can stimulate smooth muscle contraction in the prostate and
urethra, obstructing urinary flow through the urethra.
b. Anticholinergic drugs (urinary and GI antispasmodics, antihistamines, TCAs, phenothiazines)
can reduce the ability of the bladder detrusor muscle to contract and empty the bladder.
c. Diuretics can increase urinary frequency and volume.
d. Testosterone replacement can stimulate prostate growth.

D. Treatment of BPH
1. α-Adrenergic blockers: Relieve LUTS in men with AUASI scores that are moderate or severe by
reducing smooth muscle contraction in the urethra and surrounding tissues.
a. Nonspecific α-adrenergic blockers such as doxazosin and terazosin also lower BP. They require
dose titration and are not usually effective at lower doses.
b. Newer agents are selective antagonists of α1-adrenergic receptors (tamsulosin, silodosin) and
selective antagonists of postsynaptic α1-adrenergic receptors (alfuzosin) in prostate, bladder base,
bladder neck, prostatic capsule, and prostatic urethra tissues.
c. All the α-blockers can cause hypotension.
d. Compared with placebo, the α-blockers have been shown to be effective in improving LUTS in
patients with BPH. A 4- to 6-point reduction in the AUASI can be expected.
e. Differences between the agents are related to their adverse effect profile. Although the newer
agents are more specific antagonists, all agents can cause hypotension.
f. All α-blockers are metabolized through the CYP3A4 pathway and have drug interactions with
strong 3A4 inhibitors and inducers.
g. Intraoperative floppy iris syndrome is a concern with α-blockers, especially tamsulosin. Men
with LUTS being offered α-blockers should be asked about planned cataract surgery. Men with
planned cataract surgery should avoid the initiation of α-blockers until their cataract surgery is
completed.
2. α-Reductase inhibitors
a. DHT is a hormone that stimulates prostate growth.
b. α-Reductase inhibitors prevent the conversion of testosterone to DHT; these agents have been
shown to modify the disease course and may reduce the risk of urinary retention and surgical
interventions.
i. Finasteride competitively inhibits type II 5-α-reductase and lowers prostatic DHT by 80%–90%.
ii. Dutasteride is a nonselective inhibitor of both type I and II 5-α-reductase. Prostatic DHT
production is quickly suppressed with this agent.
iii. Despite these pharmacologic differences, no differences between these two agents were
observed in trials; both reduce prostate size.

ACCP Updates in Therapeutics® 2012: The Pharmacotherapy Preparatory Review and Recertification Course

1-54
Geriatrics

c. α-Reductase inhibitors do not immediately reduce LUTS and should be reserved for use in men
with large prostate volume (more than 40 g). At least 6 months of therapy is usually needed to
achieve clinical benefit. Prostate size may be reduced by about 25% in this time frame.
d. PSA concentrations are used to monitor for prostate cancer. Because these agents lower PSA
concentrations, a baseline PSA test is recommended before initiating treatment with α-reductase
inhibitors.
e. Long-term therapy with an α-reductase inhibitor can increase the risk of high-grade tumors of the
prostate in healthy men without a history of prostate cancer.
3. Phosphodiesterase (PDE)-5 inhibitors
a. Tadalafil 5 mg once daily is approved for use in BPH.
b. Mechanism thought to be caused by PDE-induced smooth muscle relaxation in the bladder,
urethra, and prostate
c. Studied as monotherapy; FDA does not recommend in combination with α-blockers because
the combination has not been adequately studied for the treatment of BPH, and there is a risk
of lowering BP. However, this combination is sometimes used in practice to treat both BPH and
erectile dysfunction with a 4-hour separation of doses.
4. Combination therapy
a. May be needed in men with LUTS, a larger prostate size, and an elevated PSA
b. Finasteride and doxazosin most studied; dutasteride is FDA label approved for use with
tamsulosin in symptomatic men having an enlarged prostate
c. Two large clinical trials (MTOPS and CombAT) have evaluated monotherapy versus combination
therapy and have concluded that, in men with LUTS and an enlarged prostate, further benefit can
be achieved by the use of the two drugs in combination.
5. Supplements
a. Saw palmetto plant extract (Serenoa repens)
b. Conflicting evidence regarding the efficacy of saw palmetto in relieving LUTS; two recent trials
suggested no benefit over placebo
c. Use of this agent with 5-α-reductase inhibitors may reduce the efficacy of the reductase inhibitors.
6. Surgery is preferred in men with severe symptoms and in those with moderate symptoms who have
not adequately responded to medical options.
7. Anticholinergic agents can be appropriate and effective treatment alternatives in men without an
elevated PVR when LUTS are predominantly storage (irritative) symptoms.

Patient Case
12. A.W. is an 85-year-old man who presents to his physician with LUTS. A digital rectal examination confirms
the diagnosis of BPH, and the physician schedules a further workup including a prostate ultrasound, which
indicates his prostate volume is 31 g. A.W.’s score on the AUASI is 15. His BP is 118/70 sitting, 102/62 stand-
ing. Which one of the following therapies is best at this time?
A. Terazosin.
B. Finasteride.
C. Tamsulosin.
D. Finasteride plus tamsulosin.

ACCP Updates in Therapeutics® 2012: The Pharmacotherapy Preparatory Review and Recertification Course

1-55
Geriatrics

Table 10. Comparison of Drugs for the Treatment of Benign Prostatic Hypertrophy
Medication Dose Range Adverse Effects Comments
Terazosin/ 1–10 mg PO QHS Orthostatic hypotension Initiate at low dose; can titrate
Doxazosin 1–8 mg PO QHS up every 2–7 days
Alfuzosin 10 mg PO DAILY Orthostatic hypotension No need to titrate
Tamsulosin 0.4–0.8 mg PO QHS May cause less orthostasis, but has Recommended for patients who
higher rate of ejaculatory dysfunction cannot tolerate α1-blockers
Silodosin 8 mg PO daily Similar to tamsulosin in causing Contraindicated if CrCl < 30
4 mg PO daily if CrCl ejaculatory dysfunction; appears less mL/minute
30–50 mL/minute sedating
Finasteride/ 5 mg PO daily Decreased libido Onset of action is usually 6
Dutasteride 0.5 mg PO daily months
CrCl = creatinine clearance; PO = orally; QHS = every night at bedtime.

VI. Arthritis

A. Osteoarthritis
1. Epidemiology
a. OA is the most prevalent form of arthritis, affecting more than 46 million Americans.
b. Highly associated with aging
c. Women are afflicted more often than men and often live to old age with disability from OA.
d. Large weight-bearing joints, such as the hip and knee, are commonly affected.
2. Etiology and pathophysiology
a. Several concomitant conditions can greatly increase the likelihood of developing OA.
i. Obesity
ii. Joint damage from repetitive movements and certain sports
iii. Genetics and family history
b. Loss of cartilage occurs in the joint as the balance of chondrocyte function shifts from formation
to destruction.
c. Subchondral bone is damaged, and the joint space narrows.
d. Symptoms of pain result from activation of nociceptive nerve endings within the damaged joint.

B. Therapy goals are to relieve pain, maintain or improve joint function, prevent loss of function, and
maintain or improve quality of life.

C. Nonpharmacologic Treatment
1. Patients need education to understand the chronic nature of OA.
2. Weight loss will decrease the biomechanical load on large weight-bearing joints; even small amounts
of weight loss help decrease pain and disability.
3. Physical and occupational therapy
4. Exercise
5. Surgery

D. Drug Therapy (analgesics are summarized in a table at the end of the Rheumatoid Arthritis section)
1. Acetaminophen is the analgesic of choice to treat pain from OA. Early research in the treatment of OA
of the knee recommended 1 g taken 4 times/day.

ACCP Updates in Therapeutics® 2012: The Pharmacotherapy Preparatory Review and Recertification Course

1-56
Geriatrics

a. The FDA estimates 400 people die every year of liver failure related to unintentional
acetaminophen overdose.
b. Patients often take different combination products containing acetaminophen, leading to
excessive dosages.
c. The FDA has changed the available OTC products, so only 325-mg tablets are available.
d. The new recommended maximal daily dosage is 3000 mg/day.
e. Individuals with liver disease should receive doses of acetaminophen that are less than 2600 mg/day.
2. In older people with OA, NSAIDs should seldom be used.
a. In selected patients, cyclooxygenase-2 (COX-2) inhibitors and NSAIDs may be used with extreme
caution when other therapies have failed and when the benefit of treatment outweighs the risk.
b. For those using a nonselective NSAID such as ibuprofen, naproxen, indomethacin, and
diclofenac, some authorities recommend that a proton pump inhibitor be used for gastric
protection.
c. Histamine-2 blockers can be considered, but in studies, they have been shown to be inferior to
proton pump inhibitors in preventing ulcer formation and bleeding.
d. For patients taking any NSAID and aspirin (for cardiac disease), a proton pump inhibitor may be
recommended for gastric protection, provided the patient is not taking clopidogrel. Patients should
be educated to take their aspirin at least 30 minutes before their first daily dose of ibuprofen or
other NSAID in the morning to avoid any interaction and reduction in aspirin efficacy.
e. Older patients receiving treatment with NSAIDs or COX-2 inhibitors should be closely monitored
for gastric and renal toxic reactions, hypertension, heart failure, and drug-drug interactions.
3. Opioids
a. Patients with persistent pain from OA that is moderate or severe are candidates for treatment with
opioids. The American Geriatrics Society recommends treatment with opioids for OA when older
patients do not respond to initial therapy with acetaminophen.
b. Monitor and anticipate opioid adverse effects and treat accordingly.
c. Do not exceed acetaminophen daily dosage guidelines when using combination products.
d. Patients should be reassessed in an ongoing fashion to ensure that treatment goals are being met.
4. Adjuvant drug therapy
a. Use appropriate medications, such as gabapentin, in patients with neuropathic pain from OA.
b. Avoid the use of tricyclic antidepressants in older patients because of the increased risk of
anticholinergic adverse effects.
c. Avoid the use of skeletal muscle relaxants such as cyclobenzaprine and carisoprodol because
of the high risk of anticholinergic adverse effects and lack of efficacy. In patients with muscle
spasms related to OA, the use of baclofen could be considered.
d. There is limited efficacy for topical agents such as capsaicin, diclofenac gel, and lidocaine patch
5% for OA.
e. Short-term relief of pain may be achieved with intra-articular hyaluronic acid or
glucocorticosteroid injections.
5. Alternative dietary supplements: Glucosamine sulfate, 500 mg taken 3 times/day, with or without
chondroitin, may be considered for chronic therapy to prevent joint degradation and relieve pain.
a. Evidence to support treatment is contradictory.
b. The adverse effect profile of glucosamine is similar to that of placebo and includes
gastrointestinal complaints.

ACCP Updates in Therapeutics® 2012: The Pharmacotherapy Preparatory Review and Recertification Course

1-57
Geriatrics

Patient Case
13. W.F. is an 85-year-old man who presents to his physician with pain from hip OA. He also has hypertension,
coronary artery disease, and BPH. For his OA, W.F. has been taking acetaminophen 650 mg 3 times/day.
W.F. reports that acetaminophen helps, but he still experiences pain that limits his ability to walk. Which
one of the following is the best next step in analgesic therapy for W.F.?
A. Change the analgesic to celecoxib.
B. Add hydrocodone.
C. Change the analgesic to ibuprofen.
D. Add glucosamine.

VII. Rheumatoid Arthritis

A. Epidemiology
1. A systemic disease characterized by a bilateral inflammatory arthritis that usually affects the small
joints of the hands, wrists, and feet
2. The prevalence is estimated to be between 1% and 2%, with women afflicted 3 times more often than men.
3. Rheumatoid arthritis can occur at any age; it is often seen in younger people.
4. Rheumatoid arthritis is an autoimmune disease with a strong genetic predisposition.

B. Pathophysiology and Clinical Presentation


1. Chronic inflammation of the synovium leads to proliferation and the development of a pannus.
2. The pannus invades joint cartilage and eventually causes erosion of the bone as well as joint destruction.
3. The cause of the initial inflammatory activation is unknown, but once activated, the immune system
produces antibodies and cytokines that accelerate cartilage and joint destruction.
4. Patients present with joint pain and stiffness, fatigue, and other signs of chronic disease. Symptoms
also include warmth, redness, and swelling of the joints.
5. Laboratory tests often reveal a positive rheumatoid factor (RF), elevated sedimentation rate, C-reactive
protein, and normochromic normocytic anemia.
6. Rheumatoid arthritis can also affect other organs such as the lung.

C. Treatment
1. The treatment goal is to control the inflammatory process so that disease remission occurs. This should
lead to relief of pain, maintenance of function, and improved quality of life.
a. Measurement of treatment response:
i. Reduction in the number of affected joints and in joint tenderness and swelling
ii. Improvement in pain
iii. A decreased amount of morning stiffness
iv. Reduction in serologic markers such as RF
v. Improvement in quality-of-life scales
2. Disease-modifying antirheumatic drugs (DMARDs)
a. DMARD therapy should be initiated within the first few months of diagnosis.
i. Methotrexate, hydroxychloroquine, sulfasalazine, and leflunomide are commonly used as
first-line agents.
ii. Biologic agents that inhibit tumor necrosis factor and interleukin receptor antagonists are
often used in patients who do not respond to a first-line agent such as methotrexate.
iii. DMARDs generally require about 3 months of use before an effect is seen.

ACCP Updates in Therapeutics® 2012: The Pharmacotherapy Preparatory Review and Recertification Course

1-58
Geriatrics

b. NSAIDs and/or glucocorticosteroids should be used for the immediate treatment of pain and
inflammation.
i. NSAIDs do not affect disease progression in rheumatoid arthritis; their anti-inflammatory
effect is seen within 1–2 weeks of daily dosing, whereas the analgesic effect begins within
several hours of administration.
ii. Glucocorticosteroids are not recommended for long-term use because of their high number
of adverse effects and long-term complications. They are often used as bridge therapy to
provide anti-inflammatory effects while waiting for the DMARDs to take effect.
c. The order of use of disease-modifying therapy is unclear. Often, methotrexate is used as first-
line therapy. If the treatment response is not optimal, combination DMARDs may be tried, or a
biologic agent can be added to methotrexate.
d. Published guidelines provide summary information to help guide therapy. Patients are
categorized by their history of disease response and severity of disease indicators to determine
the appropriate use of biologic agents.
3. Nonpharmacologic treatment
a. Rest during periods of disease exacerbation
b. Occupational and physical therapy to support mobility and maintain function
c. Maintenance of a normal weight (avoid overweight and obesity) to reduce biomechanical stress
on joints

Patient Case
14. F.A. is a 55-year-old woman with rheumatoid arthritis. On diagnosis 1 year ago, F.A. had an RF titer of 1:64,
signs and symptoms of inflammation in the joints of both hands, and about 45 minutes of morning stiffness.
She began therapy with methotrexate, and she is presently receiving 15 mg every week, folic acid 2 mg/day,
ibuprofen 800 mg 3 times/day, and omeprazole 20 mg/day. At today’s clinic visit, F.A. reports a recurrence
of her symptoms. Radiographic evaluation of her hand joints shows progression of joint space narrowing
and bone erosion. Which one of the following is the best next step in therapy for F.A.?
A. Administer etanercept.
B. Switch to hydroxychloroquine.
C. Add prednisone bridge therapy.
D. Change to leflunomide.

ACCP Updates in Therapeutics® 2012: The Pharmacotherapy Preparatory Review and Recertification Course

1-59
Geriatrics

Table 11. Selected Drug Therapy for Osteoarthritis and Rheumatoid Arthritisa
Drug Starting Dose Comments
Preferred Nonopioid Analgesics for Osteoarthritis
Acetaminophen 650 mg every 6 hours First-line therapy. Maximal daily dose from all sources
should not exceed 3000 mg
Celecoxib 100 mg/day For patients also receiving antiplatelet therapy for cardiac
disease, gastric protection should be employed
Tramadol 25 mg every 4–6 hours An extended-release form is available; mixed opioid and
SSRI effects may lead to serotonin syndrome in patients
also receiving antidepressant therapy
Selected Opioid Analgesics for Osteoarthritis
Hydrocodone 2.5–5 mg every 4–6 hours Daily dose limited by the other analgesic components of
most products
Oxycodone 2.5–5 mg every 4–6 hours As above for the combination products; sustained-release
preparation is usually used every 12 hours, but dosing can
vary from every 8–24 hours
Morphine 2.5–10 mg every 4–6 hours A variety of dosage forms are available including extended-
and immediate-release forms
Hydromorphone 1–2 mg every 3–4 hours Short duration of action
Fentanyl 12- to 25-mcg/hour patch Not recommended in opioid-naive patients; peak effects
every 72 hours from the transdermal system usually occur 18–24 hours
after the first dose, with steady state achieved after 1–2
weeks of therapy
Selected NSAIDs for Rheumatoid Arthritis
Anti-inflammatory Dose
Ibuprofen 1.2–3.2 g/day Monitor for GI ulceration, bleeding, and renal toxicity;
Meloxicam 7.5–15 mg/day anti-inflammatory effect may take 1–2 weeks; should be
Nabumetone 1–2 g/day avoided in patients with chronic kidney disease or heart
Naproxen 0.5–1.5 g/day failure
Selected DMARDs for Rheumatoid Arthritis
Customary Dose
Methotrexate 7.5–15 mg every week Probably first-line DMARD; monitor for myelosuppression,
liver dysfunction, and pulmonary fibrosis; a teratogen
Leflunomide 10–20 mg/day Similar to methotrexate; an initial loading dose may give
(Arava) therapeutic response within the first month
Hydroxychloroquine 200–300 mg 2 times/day Must routinely monitor for ocular toxicity; however, this
(Plaquenil) agent has a better adverse effect profile overall
Sulfasalazine 500–1000 mg 2 times/day GI adverse effects often limit the use of this agent
Etanercept 50 mg SC weekly Binds to TNF, inactivating this cytokine; generally well
(Enbrel) tolerated; usually used in those whose methotrexate therapy
fails; monitor for infection; check baseline PPD
Infliximab 3 mg/kg IV at 0, 2, and 6 A mouse/human chimeric antibody to TNF; used in
(Remicade) weeks and then every 8 combination with methotrexate to prevent formation of
weeks thereafter antibodies to this protein; monitor for infection; check
baseline PPD

ACCP Updates in Therapeutics® 2012: The Pharmacotherapy Preparatory Review and Recertification Course

1-60
Geriatrics

Table 11. Selected Drug Therapy for Osteoarthritis and Rheumatoid Arthritisa (continued)
Drug Starting Dose Comments
Adalimumab 40 mg SC every 2 weeks Human antibody to TNF; less antigenic than other TNF
(Humira) antibodies; monitor for infection; check baseline PPD
Anakinra 100 mg SC daily IL-1 receptor antagonist; avoid combination therapy with
(Kineret) TNF agents because of increased risk of infection
Rituximab Two infusions of 1000 mg Chimeric antibody to CD20 protein on B lymphocytes;
(Rituxan) given 2 weeks apart corticosteroid infusions help reduce infusion reactions;
used in combination with methotrexate to improve response
Abatacept Weight-based dose every Inhibits interactions between antigens and T cells; may
(Orencia) 2 weeks for two doses and be useful in those who do not respond to TNF inhibitors;
then monthly (i.e., 750 mg for monitor for infusion reactions
those weighing 60–100 kg)
Golimumab 50 mg SC every month Monoclonal antibody against TNF. Intended for use in
(Simponi) combination with methotrexate. Monitor for infections
Certolizumab pegol 400 mg SC at 0, 2, and 4 Monoclonal antibody against TNF; may have best response
(Cimzia) weeks, then 200 mg every when used in combination with methotrexate. Monitor for
other week infections
Tocilizumab 4 mg/kg IV infusion every 4 Anti-human IL-6 receptor monoclonal antibody; indicated
(Actemra) weeks; can increase to 8 mg/ for patients who have not responded to TNF inhibitors.
kg based on clinical response Monitor for infections
a
Tofacitinib is an oral Janus kinase inhibitor that may be reviewed by the FDA in 2012.
DMARD = disease-modifying antirheumatic drug; GI = gastrointestinal; IL = interleukin; IV = intravenous(ly); NSAID = nonsteroidal anti-
inflammatory drug; PPD = purified protein derivative; SC = subcutaneously; SSRI = selective serotonin reuptake inhibitor; TNF = tumor
necrosis factor.

Acknowledgment: The contributions of the previous author, Dr. Norma Owens, are acknowledged.

ACCP Updates in Therapeutics® 2012: The Pharmacotherapy Preparatory Review and Recertification Course

1-61
Geriatrics

References

Principles to Promote Optimal Dementia


Medication Use in Older People 1. Feldman HH, Jacova C, Robillard A, et al. Diag-
1. Starner CI, Gray SL, Guay DRP, Hajjar ER, Han- nosis and treatment of dementia. 2. Diagnosis.
dler SM, Hanlon JT. Geriatrics. In: DiPiro JT, Tal- CMAJ 2008;178:825–36. (Part 2 of a 5-part series
bert RL, Yee GC, et al, eds. Pharmacotherapy. A on dementia).
Pathophysiologic Approach. New York: McGraw- 2. Slattum PW, Swerdlow RH, Massey Hill A. Al-
Hill, 2008:57–66. zheimer disease. In: DiPiro JT, Talbert RL, Yee GC,
2. Schwartz JB. The current state of knowledge on eds. Pharmacotherapy. A Pathophysiologic Ap-
age, sex, and their interactions on clinical pharma- proach. New York: McGraw-Hill, 2008:1051–65.
cology. Clin Pharmacol Ther 2007;82:87–96. 3. Birks J. Cholinesterase inhibitors for Alzheimer’s
3. Elliott DP. Pharmacokinetics and pharmacody- disease. Cochrane Database Syst Rev 2006;1:
namics in the elderly. In: Schumock G, Brundage CD005593.
D, Chapman M, et al, eds. Pharmacotherapy Self- 4. Gill SS, Anderson GM, Fischer HD, et al. Syncope
Assessment Program, 5th ed. Lenexa, KS: Ameri- and its consequences in patients with dementia re-
can College of Clinical Pharmacy, 2004:115–30. ceiving cholinesterase inhibitors. Arch Intern Med
4. Lamb EJ, Webb MC, Simpson DE, Coakley AJ, 2009;169:867–73.
Newman DJ, O’Riordan SE. Estimation of glomer- 5. Grimmer T, Kurz A. Effects of cholinesterase in-
ular filtration rate in older patients with chronic hibitors on behavioural disturbances in Alzheim-
renal insufficiency: is the modification of diet in er’s disease: a systematic review. Drugs Aging
renal disease formula an improvement? J Am Geri- 2006;23:957–67.
atr Soc 2003;51:1012–7. 6. Qaseam A, Snow V, Cross JT, et al. Current phar-
5. The American Geriatrics Society 2012 Beers Crite- macologic treatment of dementia: a clinical prac-
ria Update Expert Panel.  American Geriatrics Soci- tice guideline from the American College of Physi-
ety Updated Beers Criteria for Potentially Inappro- cians and the American Academy of Family Physi-
priate Medication Use in Older Adults. J Am Geri- cians. Ann Intern Med 2008;148:370–8.
atr Soc 2012; DOI:10.1111/j.1532-5415.2012.03923.x 7. American Geriatrics Society. A Guide to Dementia
6. Higashi T, Shekelle PG, Solomon DH, et al. The Diagnosis and Treatment. 2010. Available at http://
quality of pharmacologic care for vulnerable older dementia.americangeriatrics.org. Accessed Febru-
patients. Ann Intern Med 2004;140:714–20. ary 13, 2012.
7. Handler SM, Wright RM, Ruby CM, Hanlon
JT. Epidemiology of medication-related adverse Behavioral Symptoms of Dementia
events in nursing homes. Am J Geriatr Pharmaco- 1. Howard RJ, Juszczak E, Ballard CG, et al. Done-
ther 2006;4:264–72. pezil for the treatment of agitation in Alzheimer’s
8. Moore TJ, Cohen MR, Furberg CD. Serious ad- disease. N Engl J Med 2007;357:1382–92.
verse drug events reported to the Food and Drug 2. Schneider LS, Dagerman KS, Insel P. Risk of death
Administration, 1998–2005. Arch Intern Med with atypical antipsychotic drug treatment for de-
2007;167:1752–9. mentia. Meta-analysis of randomized placebo-con-
9. Summary of the Updated American Geriatrics So- trolled trials. JAMA 2005;294:1934–43.
ciety/British Geriatrics Society Clinical Practice 3. Wang PS, Schneeweiss S, Avorn J, et al. Risk of
Guideline for Prevention of Falls in Older Persons. death in elderly users of conventional vs. atypi-
J Am Geriatr Soc 2011;59:148–57. cal antipsychotic medications. N Engl J Med
10. ACCF/AHA 2011 Expert Consensus Document on 2005;353:2335–41.
Hypertension in the Elderly. A Report of the Amer- 4. Schneider LS, Tariot PN, Dagerman KS, et al.
ican College of Cardiology Foundation Task Force Effectiveness of atypical antipsychotic drugs in
on Clinical Expert Consensus Documents. J Am patients with Alzheimer’s disease. N Engl J Med
Coll Cardiol 2011;57:2037–114. 2006;355:1525–38.

ACCP Updates in Therapeutics® 2012: The Pharmacotherapy Preparatory Review and Recertification Course

1-62
Geriatrics

5. Yury CA, Fisher JE. Meta-analysis of the effective- 6. Roehrborn CG, Siami P, Barkin J, et al. The ef-
ness of atypical antipsychotics for the treatment of fects of dutasteride, tamsulosin and combination
behavioural problems in persons with dementia. therapy on lower urinary tract symptoms in men
Psychother Psychosom 2007;76:213–8. with benign prostatic hyperplasia and prostatic en-
6. Sink KM, Holden KF, Yaffe K. Pharmacologi- largement: 2-year results from the CombAT study.
cal treatment of neuropsychiatric symptoms J Urol 2008;179:616–21.
of dementia. A review of the evidence. JAMA 7. American Urologic Association. Clinical Guide-
2005;293:596–8. lines. Management of benign prostatic hyperpla-
7. Ballard C, Hanney ML, Theodoulou M, et al. The sia. 2010. Available at www.auanet.org/content/
dementia antipsychotic withdrawal trial (DART- guidelines-and-quality-care/clinical-guidelines.
AD): long-term follow-up of a randomized place- cfm?sub=bph. Accessed February 13, 2012.
bo-controlled trial. Lancet Neurol 2009;8:151–7. 8. Sherman JJ. Health issues in older men. In: Dun-
[Epub 2009 Jan 8] sworth T, Richardson M, Chant C, et al, eds. Phar-
8. Ballard CG, Waite J, Birks J. Atypical anti- macotherapy Self-Assessment Program, 6th ed.
psychotics for aggression and psychosis in Al- Women’s and Men’s Health. Lenexa, KS: Ameri-
zheimer’s disease. Cochrane Database Syst Rev can College of Clinical Pharmacy, 2008:163–80.
2006;1:CD003476.
9. Sutor B, Rummans TA, Smith GE. Assessment and Osteoarthritis and Rheumatoid Arthritis
management of behavioral disturbances in nurs- 1. Pharmacological management of persistent pain in
ing home patients with dementia. Mayo Clin Proc older persons. J Am Geriatr Soc 2009;57:1331–46.
2001;76:540–50.
2. ACR Subcommittee on Osteoarthritis Guidelines.
10. Maher AR, Maglione M, Bagley S, et al. Efficacy Recommendations for the medical management of
and comparative effectiveness of atypical anti- osteoarthritis of the hip and knee: 2000 update. Ar-
psychotic medications for off-label uses in adults. thritis Rheum 2000;43:1905–15.
JAMA 2011;306:1359–69.
3. U.S. Food and Drug Administration. Acetamino-
phen Information. Available at www.fda.gov/
Urinary Incontinence and Benign Drugs/DrugSafety/InformationbyDrugClass/
Prostatic Hypertrophy ucm165107.htm. Accessed February 13, 2012.
1. Yamaguchi O, Nishizawa O, Takeda M, et al. Clin- 4. Saag KG, Teng GG, Patkar NM, et al. American
ical guidelines for overactive bladder. Int J Urol College of Rheumatology 2008 recommendations
2009;16:126–42. for the use of nonbiologic and biologic disease-
2. Chughtai B, Levin R, De E. Choice of antimus- modifying anti-rheumatic drugs in rheumatoid ar-
carinic agents for overactive bladder in the older thritis. Arthritis Rheum 2008;59:762–84.
patient: focus on darifenacin. Clin Interv Aging 5. Bruce SP. Rheumatoid arthritis. In: Richardson M,
2008;3:503–9. Chant C, Cheng JWM, et al, eds. Pharmacotherapy
3. Rogers R. Urinary stress incontinence in women. Self-Assessment Program, 6th ed. Chronic Ill-
N Engl J Med 2008;358:1029–36. ness II. Lenexa, KS: American College of Clinical
Pharmacy, 2009:73–90.
4. Edwards JL. Diagnosis and management of be-
nign prostatic hyperplasia. Am Fam Physician 6. Clegg DO, Reda DJ, Harris CL, et al. Glucos-
2008;77:1403–10. amine, chondroitin sulfate, and the two in com-
bination for painful knee osteoarthritis. N Engl J
5. Kaplan SA, Roehrborn CG, McConnell JD, et al.
Med 2006;354:795–808.
Long-term treatment with finasteride results in
a clinically significant reduction in total prostate 7. McAlindon TE, LaValley MP, Gulin JP, et al. Glu-
volume compared to placebo over the full range cosamine and chondroitin for treatment of osteoar-
of baseline prostate sizes in men enrolled in the thritis: a systematic quality assessment and meta-
MTOPS trial. J Urol 2008;180:1030–3. analysis. JAMA 2000;283:1469–75.

ACCP Updates in Therapeutics® 2012: The Pharmacotherapy Preparatory Review and Recertification Course

1-63
Geriatrics

Answers and Explanations to Patient Cases

1. Answer C drug that could contribute to confusion. Anticholiner-


This patient is typical of an older person in a nurs- gics such as trihexyphenidyl can cause confusion. Be-
ing home with many chronic diseases requiring drug cause this drug is likely part of the patient’s Parkinson
management. At this time, she has several risk factors disease treatment, the dose should be slowly reduced,
for falls, including a history of fall with hip fracture; and the patient should be monitored for exacerbations
diseases such as diabetes, dementia, and hypertension; of her Parkinson disease.
dizziness; and use of several drugs. An assessment for
gait and balance would help determine the severity of 6. Answer D
her risk. Rivastigmine is a potent inhibitor of acetyl and butyl
cholinesterase, leading to significant cholinergic ad-
2. Answer D verse effects such as nausea, vomiting, and diarrhea.
Renal elimination is usually the most significantly These adverse effects can be diminished by a slow-dos-
changed pharmacokinetic parameter in older people. In age titration of the drug. In addition, when a patient is at
this patient, her advanced age and diseases will add to his or her full therapeutic dose, use of the transdermal
her loss of renal function. Using the Cockcroft-Gault delivery system will generate even plasma concentra-
equation, N.H.’s estimated creatinine clearance is 24 tions and lessen the incidence of cholinergic adverse ef-
mL/minute. fects. On the basis of the maintenance dose achieved of
Creatinine clearance = [(140 − 85)65/(72 × 1.8)] × 0.85 12 mg of rivastigmine, this man can be changed to the
18-mg patch that delivers 9.5 mg/day.
3. Answer A
The diabetes treatment should be addressed promptly. 7. Answer C
Because N.H. has considerable renal insufficiency, she Over 3 years, R.A. has declined 4 points on her MMSE,
does not meet the prescribing guidelines for metformin. which suggests a treatment response to donepezil. Fur-
Use of metformin in individuals with impaired renal thermore, R.A. is still able to live at home with her
function increases the likelihood of lactic acidosis. In husband, and she has maintained some function in her
addition, glyburide is partly eliminated in the kidney, basic ADLs. Because she has benefited from acetyl-
has duration of effect of about 24 hours, and is not rec- cholinesterase inhibitor use, it should not be abruptly
ommended for elderly patients with poor renal elimina- discontinued. Evidence from clinical trials with me-
tion. N.H. could be experiencing periods of hypoglyce- mantine shows that an additional treatment response
mia that contribute to her dizziness. can be observed when memantine is added to donepezil
therapy. Memantine should be initiated at 5 mg/day and
4. Answer A increased every 2 weeks until the full therapeutic dose
Efforts to maintain bone and muscle strength are im- is achieved (10 mg 2 times/day).
portant for N.H. Most people who are older do not con-
sume a diet rich in calcium or vitamin D. In addition, 8. Answer C
because N.H. resides in a nursing home, she will have This patient has several issues related to her medica-
less sun exposure and is more likely to be deficient in tion regimen. Patients in late stages of dementia (as evi-
vitamin D, which is a risk factor for falls and reduced denced by an MMSE of 5/30) will develop a functional
muscle strength. incontinence caused by their loss of cognition and in-
ability to recognize toileting needs. Oxybutynin is an
5. Answer C anticholinergic agent useful in treating overactive blad-
This patient has mild-moderate dementia in addition to der rather than functional incontinence, and it also has
Parkinson disease. Although some patients with Par- pharmacologic properties that oppose the action of do-
kinson disease develop dementia, many do not. This nepezil. A review of the patient’s incontinence history
patient has been stable for some years. When evaluating will help determine whether this drug is efficacious in
her cognitive loss, it is important to limit the use of any the treatment of her UI.

ACCP Updates in Therapeutics® 2012: The Pharmacotherapy Preparatory Review and Recertification Course

1-64
Geriatrics

9. Answer B 12. Answer C


Discontinue oxybutynin. This drug is classified as an Pharmacologic therapy targeted at reducing urethral
inappropriate drug on the basis of Beers consensus sphincter pressure has proven effective in reducing
criteria. In addition, oxybutynin is highly anticholin- BPH symptoms. Tamsulosin is an α-adrenergic blocker
ergic and may lead to confusion in older patients. If with more specific activity for the genitourinary sys-
the patient does have overactive bladder (rather than tem. Given that A.W. already has orthostasis, tamsu-
functional incontinence), alternative medications can losin would be preferred over terazosin for this pa-
be used that more specifically target the bladder muscle tient. Orthostatic hypotension can still occur with all
with less potential for central nervous system effects. α-adrenergic blockers, so patients should be monitored
when therapy is initiated. Finasteride, an α-reductase
10. Answer A inhibitor, and combination therapy with these agents
Hyper-vocalization is a difficult behavior to address. are recommended when there is evidence of large pros-
In general, medications are not very efficacious in this tate size.
case. Adding quetiapine would likely result in sedation
in the patient as well as add the increased risk of mor- 13. Answer B
tality seen with APs. Although the behavior might de- The AGS recommends treatment with opioids for OA
crease during periods of sedation, the behavior often re- when older patients do not respond to initial therapy
turns when the patient adjusts or develops a tolerance to with acetaminophen. The NSAIDs and COX-2 inhibi-
the sedative properties. The other drug choices, adding tors are seldom considered when a thorough assessment
valproic acid or citalopram, do not have much evidence of the patient shows that the risk of treatment (gastroin-
of effectiveness in the literature. A behavioral approach testinal bleeding and renal disease) does not outweigh
is the best method to try in this patient. Types of inter- the potential benefit. Glucosamine can be added to this
ventions that could be effective include those that create patient’s medication regimen; however, if effective, it
a soothing, serene environment for the patient, such as will not provide immediate relief of pain.
soft music. Activities appropriate to the patient’s lev-
el of cognition might also be helpful as a distracting 14. Answer A
mechanism as well as a way to improve interactions. This is an example of a young woman with indicators of
Reassurance by staff is also particularly helpful so that poor prognosis with rheumatoid arthritis (positive RF,
the patient feels more comforted – often, moving a pa- young age, many symptoms) who has not responded
tient from his or her room and closer to the nurses’ sta- to therapy with methotrexate. Although the next treat-
tion will help a patient feel less alone and less afraid. ment step is not entirely clear, her best choices would be
between combination DMARD therapy and a biologic
11. Answer C agent. Leflunomide would not be preferred because its
This patient is exhibiting symptoms of urge inconti- efficacy is similar to methotrexate. Hydroxychloro-
nence. Estrogen vaginal cream is a treatment for stress quine would not be recommended as sole therapy for
incontinence, and it would unlikely affect her UI. someone who has not responded to methotrexate. Etan-
Bethanechol is potentially an option for overflow incon- ercept has a response in 60%–75% of patients whose
tinence, but it could worsen her urge symptoms. Oxy- therapy with methotrexate has failed. Glucocorticoste-
butynin is an antimuscarinic agent that might help re- roids are used as adjunctive therapy for the first several
lieve her symptoms of UI, but it would also be the most months of treatment with a disease-modifying agent.
likely option to cause adverse events and worsen her
Alzheimer disease because it is able to cross the blood-
brain barrier so easily. There is some evidence that da-
rifenacin does not worsen cognition, and it would be
preferred over oxybutynin in this patient.

ACCP Updates in Therapeutics® 2012: The Pharmacotherapy Preparatory Review and Recertification Course

1-65
Geriatrics

Answers and Explanations to Self-Assessment Questions

1. Answer B would be affected by this change. Fortunately, most drugs


Amitriptyline is highly anticholinergic and can cause are absorbed through passive diffusion, and this does not
confusion in the elderly. A worsening of this patient’s change appreciably in the older patient.
dementia could lead to confusion, but it would not
likely occur in such a short time and would not be the 5. Answer d
assessment until a full medication review and medical Appropriateness of the dose of metoprolol can be en-
workup could be performed. Although confusion could sured by evaluating patient parameters such as BP and
be a symptom of depression, the GDS appears to indi- HR; metoprolol undergoes about a 50% first-pass ef-
cate the citalopram is effective. There is no reason to fect and is metabolized by the CYP2D6 isoenzyme
repeat a urinalysis. system. Given W.G.’s estimated creatinine clearance
of 46 mL/minute, the risks and benefits of continuing
2. Answer B metformin would need to be weighed; however, if ac-
All the choices presented for this question might re- tion were taken, a discontinuation of the drug would be
duce the risk of falls in J.T. However, Answer B is best recommended rather than a dosage reduction. Glipizide
because a great deal of literature documents the asso- is also eliminated by metabolism. Gabapentin, Answer
ciation between the use of benzodiazepines with long D, is correct because this drug is eliminated renally. On
elimination half-lives and hip fracture. Furthermore, no the basis of this patient’s creatinine clearance, his dos-
age should not exceed 1400 mg/day.
diagnosis exists that warrants a benzodiazepine for J.T.
If a diagnosis were added that required a benzodiaz-
6. Answer B
epine, a better choice would be lorazepam. Parkinson
The use of several medications is the strongest variable
disease is a risk factor for falls. Exercise can reduce the
associated with adverse drug reactions. Although one
likelihood of falls. Avoiding postural hypotension from
might suspect that older patients are more sensitive to
antihypertensive medications will also help reduce
drugs because of the pharmacokinetic and pharmaco-
falls. However, of all four choices, use of diazepam is
dynamic changes that occur with aging, age is not as
inappropriate because of its pharmacokinetic profile
important as polypharmacy when the data showing a
and lack of indication.
correlation between age and adverse drug reactions are
controlled for the number of drugs taken.
3. Answer C
Although we do not know the specific type of J.T.’s 7. Answer C
incontinence that requires tolterodine treatment, her The FDA has reduced the amount of acetaminophen in
description of her present symptoms suggests that she combination products and in the over-the-counter avail-
cannot ambulate well enough to get to the bathroom. able doses of acetaminophen. About 400 deaths occur
Her ambulation problems could be attributable to wors- annually because of unintentional overdose with acet-
ening Parkinson disease. This type of incontinence is aminophen because many consumers do not know the
referred to as “functional.” One intervention that could ingredients of combination drug products. W.G. could
help J.T. with her functional incontinence is the adop- easily be consuming more than the current recommend-
tion of a toileting program so that she voids on a sched- ed maximal dose of 3000 mg of acetaminophen daily.
ule. This would help reduce her need to ambulate to the
bathroom because of an urge to void. 8. Answer A
Tylenol PM contains diphenhydramine, a strong anti-
4. Answer A cholinergic antihistamine. Anticholinergic medications
Older patients have several age-related physiologic can block bladder contractions, which can lead to both
changes in the gastrointestinal tract that can alter absorp- an inability to void and increased PVR. This potential
tion of medications. A decrease in stomach acidity occurs, adverse effect is of particular concern in a man with
and drugs that require an acidic environment to dissolve BPH who may have some baseline amount of outlet ob-
and/or be absorbed will be affected. In this question, iron struction because of an enlarged prostate gland.

ACCP Updates in Therapeutics® 2012: The Pharmacotherapy Preparatory Review and Recertification Course

1-66
Geriatrics

9. Answer A 14. Answer B


The best evidence for use of the cholinesterase inhibi- L.M. is 95 years old, so extra caution must be taken in
tors is in patients with mild-moderate Alzheimer dis- prescribing analgesics when the risk of treatment may
ease because they can often help slow disease progres- exceed the benefit. Acetaminophen is recommended as
sion. A patient scoring a 10/30 on the MMSE would first-line therapy for the treatment of OA, and a lower
be classified as having severe dementia, where benefit dose should be used in this patient. Nonsteroidal anti-
would not be as pronounced. The NPI helps assess psy- inflammatory drugs are not recommended because of
chiatric symptoms in a patient. The data are conflicting the increased risk of gastrointestinal bleeding and renal
about whether cholinesterase inhibitors benefit a patient toxicity. L.M. is receiving aspirin 81 mg/day, which is
with neuropsychiatric symptoms. another precaution to the use of NSAIDs in older pa-
tients.
10. Answer D
Cholinesterase inhibitors prevent the breakdown of 15. Answer C
acetylcholine, resulting in increased cholinergic activ- Although prednisone is not a long-term solution to treat
ity. The initiation of cholinesterase inhibitors in older S.C.’s rheumatoid arthritis, it would be appropriate to
people with dementia can worsen incontinence through treat her pain for the short term. She would not likely
the increase in acetylcholine stimulation of the bladder. be a candidate for etanercept so close to her hospitaliza-
This type of incontinence would be classified as urge tion for sepsis. Nonsteroidal anti-inflammatory drugs
incontinence. should be avoided given her renal function. Patients re-
ceiving golimumab have not responded to other thera-
11. Answer C pies, and the drug is to be used in combination with
There is a small, but consistently increased, risk of methotrexate.
death in older patients when APs are initiated. Analy-
sis of the literature that documents this risk shows that
death usually occurs within the first 6 months of pre-
scribing an AP and is attributable to many causes. The
increased risk has been observed with all the APs. This
has led to black box labeling requirements by the FDA.

12. Answer A
Several medications can often increase sedation and
somnolence in older people. In L.M., lorazepam and
quetiapine have the greatest likelihood of causing seda-
tion. There is no indication for quetiapine, and an at-
tempt to titrate this drug down and discontinue it should
be made. In the future, the same attempt should be made
with lorazepam because the goal should be to treat
L.M.’s generalized anxiety disorder with citalopram.

13. Answer d
Patients should be assessed for reversible causes of be-
havior when a change in their status occurs. This evalu-
ation should include assessment of the patient for signs
and symptoms of hunger, dehydration, depression, pain,
delirium, sleep deprivation, infection, and drug adverse
effects. Any reversible causes for the combative behav-
ior should be corrected and a behavioral approach im-
plemented to achieve proper hygiene and personal care.

ACCP Updates in Therapeutics® 2012: The Pharmacotherapy Preparatory Review and Recertification Course

1-67
Geriatrics

ACCP Updates in Therapeutics® 2012: The Pharmacotherapy Preparatory Review and Recertification Course

1-68

Potrebbero piacerti anche